Sequence of integrable functions (f_n) conv. to f

  • Thread starter Thread starter fishturtle1
  • Start date Start date
  • Tags Tags
    Functions Sequence
Click For Summary
SUMMARY

The discussion focuses on the integrability of a limit function \( f \) derived from a sequence of integrable functions \( f_n \) on the interval \([a,b]\). It establishes that if \( f_n \) converges uniformly to \( f \), then \( f \) is integrable on \([a,b]\) by demonstrating that the upper and lower sums converge appropriately. The discussion references the measure-theoretic criterion for Riemann integrability, emphasizing that a function is Riemann-integrable if it is bounded and has a set of discontinuities with Lebesgue measure zero. The conclusion asserts that the limit function \( f \) inherits integrability from the sequence \( f_n \).

PREREQUISITES
  • Understanding of Riemann integration and its properties
  • Familiarity with uniform convergence of functions
  • Knowledge of measure theory concepts, particularly Lebesgue measure
  • Ability to work with upper and lower sums in integration
NEXT STEPS
  • Study the properties of uniform convergence in the context of integrability
  • Learn about the Riemann-Lebesgue theorem and its implications for integrability
  • Explore examples of functions that are Riemann-integrable versus those that are not
  • Investigate the relationship between Riemann and Lebesgue integrals
USEFUL FOR

Mathematicians, students of real analysis, and anyone interested in the foundations of integration theory and the properties of convergent sequences of functions.

fishturtle1
Messages
393
Reaction score
82
Homework Statement
Let ##(f_n)## be a sequence of integrable functions on ##[a,b]##, and suppose ##f_n \rightarrow f## uniformly on ##[a,b]##. Prove that ##f## is integrable and
$$\int_a^b f = \lim_{n\rightarrow\infty} \int_a^b f_n$$
Relevant Equations
.
##\textbf{Attempt at solution}##: If I can show that ##f## is integrable on ##[a,b]##, then for the second part I get :

Let ##\frac{\varepsilon}{b-a} > 0##. By definition of uniform convergence, there exists ##N = N(\varepsilon) > 0## such that for all ##x \in [a,b]## we have ##\vert f(x) - f_n(x) \vert < \frac{\varepsilon}{b-a}##. This gives us,

$$\vert \int_a^b f(x)dx - \int_a^b f_n(x)dx \vert = \vert \int_a^b f(x) - f_n(x) dx \vert \le \int_a^b \vert f(x) - f_n(x) \vert dx < \int_a^b \frac{\varepsilon}{b-a}dx = \varepsilon$$ when ##n > N##.

It follows ##\lim_{n\rightarrow\infty} \int_a^b f_n(x) dx = \int_a^b f(x) dx## for all ##x \in [a,b]##. []

For the first part I am stuck. Let ##\varepsilon > 0##. Then I need a partition ##P## of ##[a,b]## such that ##U(f, P) - L(f, P) < \varepsilon##. We're given that for any ##n##, there is a partition ##P## such that ##U(f_n, P) - L(f_n, P) < \varepsilon##. How can I proceed?
 
  • Like
Likes Delta2
Physics news on Phys.org
fishturtle1 said:
For the first part I am stuck. Let ##\varepsilon > 0##. Then I need a partition ##P## of ##[a,b]## such that ##U(f, P) - L(f, P) < \varepsilon##. We're given that for any ##n##, there is a partition ##P## such that ##U(f_n, P) - L(f_n, P) < \varepsilon##. How can I proceed?

The upper and lower sums work like integrals in a way, so you can use a similar technique for these as you used to show part b).
 
  • Informative
Likes fishturtle1
Here is a measure theoretic solution which you may appreciate later (maybe you treat this theorem in a first course, probably not).

We use the following result (see any text on measure theory or more advanced texts on Riemann-integration).
A function is Riemann-integrable if and only if it is bounded and its set of discontinuities has (Lebesgue) measure ##0##.

The points where atleast one ##f_n## is discontinuous has measure zero so the limit of the ##f_n's## is almost surely continuous as uniform convergence preserves continuity. Hence the limit is integrable.
 
Last edited by a moderator:
  • Like
Likes fishturtle1
PeroK said:
The upper and lower sums work like integrals in a way, so you can use a similar technique for these as you used to show part b).
We'll show ##f## is integrable on ##[a,b]##.

Proof: Let ##\frac{\varepsilon}{4(b-a)} > 0##. Then there is ##N = N(\frac{\varepsilon}{4(b-a)}) > 0## such that for ##x \in [a,b]## and ##n > N## implies $$\vert f_n(x) - f(x) \vert < \frac{\varepsilon}{4(b-a)}$$ i.e
$$-\frac{\varepsilon}{4(b-a)} < f(x) - f_n(x) < \frac{\varepsilon}{4(b-a)} $$
Fix ##n > N##. For ##\frac{\varepsilon}{2} > 0##, there exists partition ##P = \lbrace a = t_0 < t_1 < \dots < t_k = b \rbrace## such that ##U(f_n, P) - L(f_n, P) < \frac{\varepsilon}{2}##.

Now, $$U(f - f_n, P) = \sum_{i=1}^{k} M(f - f_n, [t_{i-1}, t_i]) \cdot (t_i - t_{i-1}) < \sum_{i=1}^{k} \frac{\varepsilon}{4(b-a)} \cdot (t_i - t_{i-1}) = \frac{\varepsilon}{4(b-a)} \sum_{i=1}^{k} (t_i - t_{i-1}) = \frac{\varepsilon}{4}$$

Also, $$L(f - f_n, P) = \sum_{i=1}^{k}m(f - f_n, [t_{i-1}, t_i])\cdot (t_i - t_{-1}) > \sum_{i-1}^{k} -\frac{\varepsilon}{4(b-a)}\cdot(t_i - t_{i-1}) = - \frac{\varepsilon}{4}$$

Thus, $$U(f, P) - L(f, P) \le U(f - f_n, P) + U(f_n, P) - (L(f - f_n, P) + L(f_n, P)) < \varepsilon$$
It follows that ##f## is integrable on ##[a,b]##. []
 
Last edited:
  • Like
Likes Delta2
Math_QED said:
Here is a measure theoretic solution which you may appreciate later (maybe you treat this theorem in a first course, probably not).

We use the following result (see any text on measure theory or more advanced texts on Riemann-integration.The points where atleast one ##f_n## is discontinuous has measure zero so the limit of the ##f_n's## is almost surely continuous as uniform convergence preserves continuity. Hence the limit is integrable.
Thanks for posting this. I think I am learning about this, this upcoming semester.
 
  • Like
Likes member 587159
fishturtle1 said:
Thanks for posting this. I think I am learning about this, this upcoming semester.

It's a very powerful tool and actually weird it doesn't occur in much places. For example, it immediately implies that continuous functions, monotonic functions, step-functions, functions with countably many discontinuities (e.g. https://en.wikipedia.org/wiki/Thomae's_function) are all Riemann-integrable. It can also be used to quickly show that something is not Riemann-integrable. For example, an indicator function on the rationals or the Cantor set can not be Riemann-integrable (note that these become integrable if one considers the Lebesgue integral instead of the Riemann-integral).
 
  • Informative
Likes fishturtle1
Question: A clock's minute hand has length 4 and its hour hand has length 3. What is the distance between the tips at the moment when it is increasing most rapidly?(Putnam Exam Question) Answer: Making assumption that both the hands moves at constant angular velocities, the answer is ## \sqrt{7} .## But don't you think this assumption is somewhat doubtful and wrong?

Similar threads

  • · Replies 6 ·
Replies
6
Views
3K
  • · Replies 8 ·
Replies
8
Views
2K
  • · Replies 4 ·
Replies
4
Views
2K
  • · Replies 12 ·
Replies
12
Views
2K
  • · Replies 7 ·
Replies
7
Views
2K
  • · Replies 7 ·
Replies
7
Views
2K
  • · Replies 15 ·
Replies
15
Views
2K
  • · Replies 7 ·
Replies
7
Views
3K
  • · Replies 9 ·
Replies
9
Views
2K
  • · Replies 7 ·
Replies
7
Views
3K